Endocrine Disorders

Ace your homework & exams now with Quizwiz!

The 68-year-old client diagnosed with hyperthyroidism is being treated with radio active iodine therapy. Which interventions should the nurse discuss with the client? 1. Explain it will take up to a month for symptoms of hyperthyroidism to subside. 2. Teach the iodine therapy will have to be tapered slowly over one (1) week. 3. Discuss the client will have to be hospitalized during the radioactive therapy. 4. Inform the client after therapy the client will not have to take any medication.

**1. Radioactive iodine therapy is used to destroy the overactive thyroid cells. After treatment, the client is followed closely for three (3) to four (4) weeks until the euthyroid state is reached.** 2. A single dose of radioactive iodine therapyis administered; the dosage is based on theclient's weight. 3. The colorless, tasteless radioiodine is administered by the radiologist, and theclient may have to stay up to two (2) hoursafter the treatment in the office. 4. If too much of the thyroid gland is destroyed by the radioactive iodine therapy, the client may develop hypothy-roidism and have to take thyroid hormonethe rest of his or her life

The nurse is developing a plan of care for the client diagnosed with acquired immunodeficiency syndrome (AIDS) who has developed an infection in the adrenal gland. Which client problem is highest priority? 1. Altered body image. 2. Activity intolerance. 3. Impaired coping. 4. Fluid volume deficit.

4 1. Altered body image is a psychosocial problem, which is not a priority over a potentially lethal physical complication, and physical changes occur over an extended period. 2. Activity intolerance will occur with adrenal gland hypofunction, but this is not a priority over dehydration. 3. Impaired coping can occur in clients with adrenal gland disorders, but it is not a priority over dehydration. **4. Fluid volume deficit (dehydration) can lead to circulatory impairment and hyperkalemia.** TEST-TAKING HINT: Assuming all of the problems listed apply to the client diagnosed with Addison's disease, two are psychosocial problems and two are physiological. Applying Maslow's hierarchy of needs, the two psychological problems can be ruled out as the highest priority. Of the two options remaining, activity intolerance is not life altering or threatening.

The client diagnosed with HHNS was admitted yesterday with a blood glucose level of 780 mg/dL. The client's blood glucose level is now 300 mg/dL. Which intervention should the nurse implement? 1. Increase the regular insulin IV drip. 2. Check the client's urine for ketones. 3. Provide the client with a therapeutic diabetic meal. 4. Notify the HCP to obtain an order to decrease insulin.

4 1. The regular intravenous insulin is continued because ketosis is not present, as with DKA. 2. The client diagnosed with type 2 diabetes does not excrete ketones in HHNS because there is enough insulin to prevent fat breakdown but not enough to lower blood glucose. 3. The client may or may not feel like eating, but it is not the appropriate intervention when the blood glucose level is reduced to 300 mg/dL. **4. When the glucose level is decreased to around 300 mg/dL, the regular insulin infusion therapy is decreased. Subcutaneous insulin will be administered per sliding scale.** TEST-TAKING HINT: When two (2) options are the opposite of each other, they can either be eliminated or can help eliminate the other two options as incorrect answers. Options "2" and "3" do not have insulin in the answer; therefore, they should be eliminated as possible answers.

The client is admitted to the ICU diagnosed with DKA. Which interventions should the nurse implement? Select all that apply. 1. Maintain adequate ventilation. 2. Assess fluid volume status. 3. Administer intravenous potassium. 4. Check for urinary ketones. 5. Monitor intake and output.

ANS: 1, 2, 3, 4, 5 1. The nurse should always address the airway when a client is seriously ill. 2. The client must be assessed for fluid volume deficit and then for fluid volume excess after fluid replacement is started. 3. The electrolyte imbalance of primary concern is depletion of potassium. 4. Ketones are excreted in the urine; levels are documented from negative to large amount. Ketones should be monitored frequently. 5. The nurse must ensure the client's fluid intake and output are equal. TEST-TAKING HINT: The test taker must select all answer options that apply. Do not try to outguess the item writer. In some instances all options are correct.

The nurse is preparing to administer A.M. medications to the following clients. Which medication should the nurse question before administering? 1. Pancreatic enzymes to the client who has finished breakfast. 2. The pain medication, morphine, to the client who has a respiratory rate of 20. 3. The loop diuretic to the client who has a serum potassium level of 3.9 mEq/L. 4. The beta blocker to the client who has an apical pulse of 68 bpm.

1 ***1. Pancreatic enzymes must be administered with meals to enhance the digestion of starches and fats in the gastrointestinal tract.** 2. The client's respiratory rate is within normal limits; therefore the morphine should be administered to the client who is having pain. 3. This is a normal potassium level; therefore the nurse would not need to question administering this medication. 4. The apical pulse is within normal limits; therefore the nurse should not question administering this medication. TEST-TAKING HINT: The test taker must determine if the assessment data provided in the answer option are abnormal, unexpected, or life-threatening so that it warrants questioning the administration of the medication. The test taker should also think about whether the administration of the medication would create an abnormal or life-threatening situation.

The nurse is administering a pancreatic enzyme to the client diagnosed with chronic pancreatitis. Which statement best explains the rationale for administering this medication? 1. It is an exogenous source of protease, amylase, and lipase. 2. This enzyme increases the number of bowel movements. 3. This medication breaks down in the stomach to help with digestion. 4. Pancreatic enzymes help break down fat in the small intestine.

1 ***1. Pancreatic enzymes enhance the digestion of starches (carbohydrates) in the gastrointestinal tract by supplying an exogenous (outside) source of the pancreatic enzymes protease, amylase, and lipase.** 2. Pancreatic enzymes decrease the number of bowel movements. 3. The enzymes are enteric coated and should not be crushed because the hydrochloric acid in the stomach will destroy the enzymes; these enzymes work in the small intestine. 4. Pancreatic enzymes help break down carbohydrates, and bile breaks down fat. TEST-TAKING HINT: Remember that enzymes break down other foods and end in "ase." The test taker must know the normal function of organs to identify correct answers.

The client is diagnosed with cancer of the head of the pancreas. When assessing the patient, which signs and symptoms would the nurse expect to find? 1. Clay-colored stools and dark urine. 2. Night sweats and fever. 3. Left lower abdominal cramps and tenesmus. 4. Nausea and coffee-ground emesis.

1 ***1. The client will have jaundice, clay-colored stools, and tea-colored urine resulting from blockage of the bile drainage.** 2. Night sweats and fevers are associated with lymphoma. 3. Left lower abdominal cramps are associated with diverticulitis, and tenesmus is straining when defecating. 4. Nausea and coffee-ground emesis are symptoms of gastric ulcers. TEST-TAKING HINT: The test taker should remember anatomical placement of organs. This would eliminate answer options "3" and "4." The pancreas empties pancreatic enzymes into the small bowel to aid in the digestion of carbohydrates and fats in close proximity to where the common bile duct enters the intestine.

The client is diagnosed with hypothyroidism. Which signs/symptoms should the nurse expect the client to exhibit? 1. Complaints of extreme fatigue and hair loss. 2. Exophthalmos and complaints of nervousness. 3. Complaints of profuse sweating and flushed skin. 4. Tetany and complaints of stiffness of the hands.

1 **1. A decrease in thyroid hormone causes decreased metabolism, which leads to fatigue and hair loss.** 2. These are signs of hyperthyroidism. 3. These are signs of hyperthyroidism. 4. These are signs of parathyroidism. TEST-TAKING HINT: Often if the test taker does not know the specific signs/ symptoms of the disease, but knows the function of the system affected by the disease, some possible answers can be ruled out. Tetany and stiffness of the hands are related to calcium, the level of which is influenced by the parathyroid gland, not the thyroid gland; therefore, option "4" can be ruled out.

The nurse is admitting a client diagnosed with syndrome of inappropriate antidiuretic hormone (SIADH). Which clinical manifestations should be reported to the health-care provider? 1. Serum sodium of 112 mEq/L and a headache. 2. Serum potassium of 5.0 mEq/L and a heightened awareness. 3. Serum calcium of 10 mg/dL and tented tissue turgor. 4. Serum magnesium of 1.2 mg/dL and large urinary output.

1 **1. A serum sodium level of 112 mEq/L is dangerously low, and the client is at risk for seizures. A headache is a symptom of a low-sodium level.** 2. This is a normal potassium level, and a heightened level of awareness indicates drug usage. 3. This is a normal calcium level and the client is fluid overloaded, not dehydrated, so there would not be tented tissue turgor. 4. This is a normal magnesium level, and a large urinary output is desired. TEST-TAKING HINT: The nurse must know common laboratory values.

The home health nurse is completing the admission assessment for a 76-year-old client diagnosed with type 2 diabetes controlled with 70/30 insulin. Which intervention should be included in the plan of care? 1. Assess the client's ability to read small print. 2. Monitor the client's serum prothrombin time (PT) level. 3. Teach the client how to perform a hemoglobin A1c test daily. 4. Instruct the client to check the feet weekly.

1 **1. Age-related visual changes and diabetic retinopathy could cause the client to have difficulty in reading and drawing up insulin dosage accurately.** 2. The PT level is monitored for clients receiving Coumadin, an anticoagulant, which is not ordered for clients with diabetes, type 1 or 2. 3. Glycosylated hemoglobin is a serum blood test usually performed in a laboratory, not in the client's home. The hemoglobin Alc is performed every three (3) months. Self-monitoring blood glucose (SMBG) should be taught to the client. 4. The client's feet should be checked daily, not weekly. In a week the client could have developed gangrene from an injury the client did not realize he or she had. TEST-TAKING HINT: Always notice the age of a client if it is provided because this is important when determining the correct answer for the question. Be sure to note the adverbs, such as "weekly" instead of "daily."

The nurse is planning the care of a client diagnosed with Addison's disease. Which intervention should be included? 1. Administer steroid medications. 2. Place the client on fluid restriction. 3. Provide frequent stimulation. 4. Consult physical therapy for gait training.

1 **1. Clients diagnosed with Addison's disease have adrenal gland hypofunction. The hormones normally produced by the gland must be replaced. Steroids and androgens are produced by the adrenal gland.** 2. The client will have decreased fluid volume, and fluid restriction exacerbates a crisis. 3. The client requires a quiet, calm, relaxed atmosphere. 4. The client walks with a stooped posture from fatigue, but gait training is not needed. TEST-TAKING HINT: To answer this question, the test taker must have knowledge of adrenal gland function.

The nurse identifies the client problem "risk for imbalanced body temperature" fort he client diagnosed with hypothyroidism. Which intervention should be included in the plan of care? 1. Discourage the use of an electric blanket. 2. Assess the client's temperature every two (2) hours. 3. Keep the room temperature cool. 4. Space activities to promote rest.

1 **1. External heat sources (heating pads,electric or warming blankets) should be discouraged because they increase the risk of peripheral vasodilation and vascular collapse.** 2. Assessing the client's temperature every two (2) hours is not needed because the temperature will not change quickly. The client needs thyroid hormones to help increase the client's temperature. 3. The room temperature should be kept warm because the client will have complaints of being cold. 4. The client is fatigued and this is an appropriate intervention, but is not applicable to the client problem of "risk for imbalanced body temperature."

The nurse administered 28 units of Humulin N, an intermediate-acting insulin, to a client diagnosed with type 1 diabetes at 1600. Which intervention should the nurse implement? 1. Ensure the client eats the bedtime snack. 2. Determine how much food the client ate at lunch. 3. Perform a glucometer reading at 0700. 4. Offer the client protein after administering insulin.

1 **1. Humulin N peaks in six (6) to eight (8) hours, making the client at risk for hypoglycemia around midnight, which is why the client should receive a bedtime snack. This snack will prevent nighttime hypoglycemia.** 2. The food intake at lunch will not affect the client's blood glucose level at midnight. 3. The client's glucometer reading should be done around 2100 to assess the effectiveness of insulin at 1600. 4. Onset of Humulin N, an intermediate-acting insulin, is two (2) to four (4) hours but it does not peak until six (6) to eight (8) hours. TEST-TAKING HINT: Remember to look at the adjective or descriptor. Intermediate-acting insulin provides the reader a clue: Anything with intermediate action, instead of longer acting, is incorrect.

Which signs/symptoms should the nurse expect to assess in the client diagnosed with an insulinoma? 1. Nervousness, jitteriness, and diaphoresis. 2. Flushed skin, dry mouth, and tented skin turgor. 3. Polyuria, polydipsia, and polyphagia. 4. Hypertension, tachycardia, and feeling hot.

1 **1. Insulinoma is a tumor of the islet cells of the pancreas that produces insulin. The signs/symptoms of an insulinoma are signs of hypoglycemia.** 2. These are signs/symptoms of hyperglycemia. 3. These are signs/symptoms of hyperglycemia. 4. These are signs/symptoms of hyperthyroidism.

The nurse identified a concept of metabolism for a client diagnosed with diabetes. Which antecedent would be identified as placing the client at risk for diabetes? 1. Nutrition. 2. Sensory perception. 3. pH regulation. 4. Medication.

1 **1. Nutrition encompasses obesity, and obesity is a risk factor for developing diabetes mellitus type 2.** 2. Sensory perception may be a problem for clients who have diabetes because ophthalmological issues occur as a result of high blood glucose levels on a prolonged period of time but are not antecedents. 3. The concept of pH is a situation that can occur as a result of DM1 but not DM2 because acidosis results from lactic acid buildup from no insulin production from the pancreas. Type 2 diabetes clients still produce some insulin. Insulin resistance is the issue in type 2 diabetes. 4. Medication is given to treat diabetes but not to cause it. TEST-TAKING HINT: The test taker must know risk factors for developing a disease process.

57. The client diagnosed with Addison's disease is admitted to the emergency department after a day at the lake. The client is lethargic, forgetful, and weak. Which intervention should the nurse implement? 1. Start an IV with an 18-gauge needle and infuse NS rapidly. 2. Have the client wait in the waiting room until a bed is available. 3. Obtain a permit for the client to receive a blood transfusion. 4. Collect urinalysis and blood samples for a CBC and calcium level.

1 **1. The client was exposed to wind and sun at the lake during the hours prior to being admitted to the emergency depart- ment. This predisposes the client to dehydration and an addisonian crisis. Rapid IV fluid replacement is necessary.** 2. Sitting in the waiting area could cause the client to go into a coma and die. 3. A blood transfusion is not an appropriate intervention for this client. 4. Laboratory specimens are not priority and calcium is not a problem in clients with Addison's disease. TEST-TAKING HINT: This client is weak, lethargic, and forgetful, indicating a diminished level of consciousness. The nurse should choose an action addressing the problem.

Which laboratory value should be monitored by the nurse for the client diagnosed with diabetes insipidus? 1. Serum sodium. 2. Serum calcium 3. Urine glucose. 4. Urine white blood cells.

1 **1. The client will have an elevated sodium level as a result of low circulating blood volume. The fluid is being lost through the urine. Diabetes means "to pass through" in Greek, indicating polyuria, a symptom shared with diabetes mellitus. Diabetes in- sipidus is a totally separate disease process.** 2. Serum calcium is not affected by diabetes insipidus. 3. Urine glucose is monitored for diabetes mellitus. 4. White blood cells in the urine indicate the presence of a urinary tract infection. TEST-TAKING HINT: The test taker should not confuse diabetes insipidus and diabetes mellitus

The elderly client is admitted to the intensive care department diagnosed with severe HHNS. Which collaborative intervention should the nurse include in the plan of care? 1. Infuse 0.9% normal saline intravenously. 2. Administer intermediate-acting insulin. 3. Perform blood glucometer checks daily. 4. Monitor arterial blood gas (ABG) results.

1 **1. The initial fluid replacement is 0.9% normal saline (an isotonic solution) intravenously, followed by 0.45% saline. The rate depends on the client's fluid volume status and physical health, especially of the heart.** 2. Regular insulin, not intermediate, is the insulin of choice because of its quick onset and peak in two (2) to four (4) hours. 3. Blood glucometer checks are done every one (1) hour or more often in clients with HHNS who are receiving regular insulin drips. 4. Arterial blood gases are not affected in HHNS because there is no breakdown of fat resulting in ketones leading to metabolic acidosis. TEST-TAKING HINT: The test taker should eliminate option "3" based on the word "daily." In the ICU with a client who is very ill, most checks are more often than daily. Remember to look at adjectives; "intermediate" in option "2" is the word eliminating this as a possible correct answer.

The client diagnosed with type 1 diabetes is found lying unconscious on the floor of the bathroom. Which intervention should the nurse implement first? 1. Administer 50% dextrose (IVP). 2. Notify the health-care provider. 3. Move the client to the ICU. 4. Check the serum glucose level.

1 **1. The nurse should assume the client is hypoglycemic and administer IVP dextrose, which will rouse the client immediately. If the collapse is the result of hyperglycemia, this additional dextrose will not further injure the client.** 2. The health-care provider may or may not need to be notified, but this is not the first intervention. 3. The client should be left in the client's room, and 50% dextrose should be administered first. 4. The serum glucose level requires a venipuncture, which will take too long. A blood glucometer reading may be obtained, but the nurse should first treat the client, not the machine. The glucometer only reads "low" after a certain point, and a serum level is needed to confirm exact glucose level. TEST-TAKING HINT: The question is requesting the test taker to select which intervention should be implemented first. All four options could be possible interventions, but only one (1) intervention should be implemented first. The test taker should select the intervention directly treating the client; do not select a diagnostic test.

The nurse and an unlicensed nursing assistant are caring for clients on an oncology floor. Which intervention should the nurse delegate to the assistant? 1. Assist the client with abdominal pain to turn to the side and flex the knees. 2. Monitor the Jackson Pratt drainage tube to make sure it is draining properly. 3. Check to see if the client is sleeping after pain medication is given. 4. Empty the bedside commode of the client who has been having melena.

1 **1. The nursing assistant can help a client to turn to the side and assume the fetal position, which would decrease some abdominal pain.** 2. This is a high-level nursing intervention that the unlicensed nursing assistant is not qualified to implement, 3. Evaluation of the effectiveness of a PRN medication must be done by the nurse. 4. The nurse should empty the bedside commode to determine if the client is continuing to pass melena (blood in the stool). TEST-TAKING HINT: There are basic rules to delegation. The nurse cannot delegate assessment, evaluation, unstable clients, or situations requiring nursing judgment.

The nurse is developing a care plan for the client diagnosed with type 1 diabetes. The nurse identifies the problem "high risk for hyperglycemia related to noncompliance with the medication regimen." Which statement is an appropriate short-term goal for the client? 1. The client will have a blood glucose level between 90 and 140 mg/dL. 2. The client will demonstrate appropriate insulin injection technique. 3. The nurse will monitor the client's blood glucose levels four (4) times a day. 4. The client will maintain normal kidney function with 30-mL/hr urine output.

1 **1. The short-term goal must address the response part of the nursing diagnosis, which is "high risk for hyperglycemia," and this blood glucose level is within acceptable ranges for a client who is noncompliant.** 2. This is an appropriate goal for a knowledge-deficit nursing diagnosis. Noncompliance is not always the result of knowledge deficit. 3. The nurse is implementing an intervention, and the question asks for a goal which addresses the problem of "high risk for hyperglycemia." 4. The question asks for a short-term goal and this is an example of a long-term goal. TEST-TAKING HINT: Remember, the nursing diagnosis consists of a problem related to an etiology. The goals must address the problem and the interventions must address the etiology. The test taker should always remember a short-term goal is usually a goal met during the hospitalization, and the long-term goal may take weeks, months, or even years.

The nurse is teaching the client diagnosed with diabetes. Which should the nurse teach to limit the complications of diabetes? 1. Teach the client to keep the blood glucose under 140 mg/dL. 2. Demonstrate how to test the urine for ketones. 3. Instruct the client to apply petroleum jelly between the toes. 4. Allow the client to eat meals as desired and then take insulin.

1 **1. To limit the complications of diabetes the client should keep the blood glucose levels under 140 mg/dL. This can be done with medications, diet, and exercise. Self glucose monitoring allows the client to monitor the glucose levels.** 2. Testing for urine ketones will not help to keep the blood glucose level controlled. 3. Petroleum jelly is rubbed on the feet but not between the toes. 4. The client should administer sliding-scale insulin when needed but not eat whatever the client wishes. The client should still attempt to control the amount of carbohydrates. TEST-TAKING HINT: The nurse must recommend measures to control or treat disease processes.

The nurse is performing discharge teaching for a client diagnosed with Cushing's disease. Which statement by the client demonstrates an understanding of the instructions? 1. "I will be sure to notify my health-care provider if I start to run a fever." 2. "Before I stop taking the prednisone, I will be taught how to taper it off." 3. "If I get weak and shaky, I need to eat some hard candy or drink some juice." 4. "It is fine if I continue to participate in weekend games of tackle football."

1 1. Cu**shing's syndrome/disease predisposes the client to develop infections as a result of the immunosuppressive nature of the disease.** 2. The client has too much cortisol; this client should not be receiving prednisone, a steroid medication. 3. These are symptoms of hypoglycemia, which is not expected in this client because this client has high glucose levels. 4. The client is predisposed to osteoporosis and fractures. Contact sports should be avoided. TEST-TAKING HINT: If the test taker is not aware of the disease problem, this question could be answered correctly because of common standard discharge instructions—namely, notify the health-care provider of a fever.

59. The nurse manager of a medical-surgical unit is asked to determine if the unit should adopt a new care delivery system. Which behavior is an example of an autocratic style of leadership? 1. Call a meeting and educate the staff on the new delivery system being used. 2. Organize a committee to investigate the various types of delivery systems. 3. Wait until another unit has implemented the new system and see if it works out. 4. Discuss with the nursing staff if a new delivery system should be adopted.

1 ** 1. An autocratic style is one in which the person in charge makes the decision without consulting anyone else.** 2. This behavior is an example of a democratic leadership style. 3. This behavior is an example of laissezfaire leadership style. 4. This behavior is an example of democratic leadership style. TEST-TAKING HINT: The test taker could choose the correct answer if the test taker knew terms such as "autocratic" and "democratic."

The nurse is teaching the client diagnosed with hyperthyroidism. Which information should be taught to the client? Select all that apply. 1. Notify the HCP if a three (3)-pound weight loss occurs in two (2) days. 2. Discuss ways to cope with the emotional lability. 3. Notify the HCP if taking over-the-counter medication 4. Carry a medical identification card or bracelet. 5. Teach how to take thyroid medications correctly.

1, 2, 3, 4, **1. Weight loss indicates the medication may not be effective and will probably need to be increased.** **2. The client needs to know emotional highs and lows are secondary to hyperthyroidism. With treatment, this emotional lability will subside.** **3. Any over-the-counter medications (for example, alcohol-based medications) may negatively affect the client's hyperthyroidism or medications being used for treatment.** **4. This will help any HCP immediately know of the client's condition, especially if the client is unable to tell the HCP.** 5. The client will be on anti thyroid medication not thyroid medication

The client with an acute exacerbation of chronic pancreatitis has a nasogastric tube and is NPO. Which interventions should the nurse implement? Select all that apply. 1. Monitor the bowel sounds. 2. Weigh the client daily. 3. Assess the intravenous site. 4. Provide oral and nasal care. 5. Monitor the blood glucose.

1, 2, 3, 4, 5 ***1. The return of bowel sounds indicates the return of peristalsis, and the nasogastric suction is usually discontinued within 24 to 48 hours thereafter. ***2. Daily weight gain reflects fluid gain. ***3. The nurse should assess for signs of infection or infiltration. ***4. Fasting and the N/G tube increase the client's risk for mucous membrane irritation and breakdown. ***5. Blood glucose levels are monitored because clients with chronic pancreatitis can develop diabetes mellitus. TEST-TAKING HINT: This alternative-type question requires the test taker to select all interventions that are appropriate for the client's diagnosis. The test taker should not try to eliminate options because all options could be correct.

Which interrelated concepts could be identified as actual or potential for a 56-year-old male client diagnosed with diabetes mellitus type 2? Select all that apply. 1. Nutrition. 2. Metabolism. 3. Infection. 4. Male reproduction. 5. Skin integrity.

1, 2, 3, 4, 5 **1. Obesity is included in the concept of nutrition. Obesity is an antecedent of diabetes mellitus type 2. **2. Diabetes is a problem of glucose metabolism. **3. The client is at greater risk for developing infections resulting from the high circulating glucose levels. Bacteria utilize glucose for energy, as do mammals. **4. Diabetes affects the ability of the blood vessels to respond to circulatory need. For a middle-aged male this can result in erectile dysfunction. **5. Skin integrity is an issue if a pressure sore or a blister occurs on the feet. If not noted and treated early then an infection can result in amputation. TEST-TAKING HINT: The test taker must know the disease process and potential complications.

The nurse is planning the care of a client diagnosed with syndrome of inappropriate antidiuretic hormone (SIADH). Which interventions should be implemented? Select all that apply. 1. Restrict fluids per health-care provider order. 2. Assess level of consciousness every two (2) hours. 3. Provide an atmosphere of stimulation. 4. Monitor urine and serum osmolality. 5. Weigh the client every three (3) days.

1, 2, 4 **1. Fluids are restricted to 500 to 600 mL per 24 hours.** **2. Orientation to person, place, and time should be assessed every two (2) hours or more often.** 3. A safe environment, not a stimulating one, is provided. **4. Urine and serum osmolality are monitored to determine fluid volume status.** 5. The client should be weighed daily. TEST-TAKING HINT: The test taker should notice numbers: Is assessing the client's level of consciousness every two (2) hours enough, or is weighing the client every three (3) days enough?

The diabetic educator is teaching a class on diabetes type 1 and is discussing sick-day rules. Which interventions should the diabetes educator include in the discussion? Select all that apply. 1. Take diabetic medication even if unable to eat the client's normal diabetic diet. 2. If unable to eat, drink liquids equal to the client's normal caloric intake. 3. It is not necessary to notify the health-care provider (HCP) if ketones are in the urine. 4. Test blood glucose levels and test urine ketones once a day and keep a record. 5. Call the health-care provider if glucose levels are higher than 180 mg/dL.

1, 2, 5 **1. The most important issue to teach clients is to take insulin even if they are unable to eat. Glucose levels are increased with illness and stress.** **2. The client should drink liquids such as regular cola or orange juice, or eat regular gelatin, which provide enough glucose to prevent hypoglycemia when receiving insulin.** 3. Ketones indicate a breakdown of fat and must be reported to the HCP because they can lead to metabolic acidosis. 4. Blood glucose levels and ketones must be checked every three (3) to four (4) hours, not daily. **5. The HCP should be notified if the blood glucose level is this high. Regular insulin may need to be prescribed to keep the blood glucose level within acceptable range.** TEST-TAKING HINT: This is an alternate-type question having more than one correct answer. The test taker should read all options and determine if each is an appropriate intervention.

The nurse is assessing a client with complaints of vague upper abdominal pain that is worse at night but is relieved by sitting up and leaning forward. Which assessment question should the nurse ask next? 1. "Have you noticed a yellow haze when you look at things?" 2. "Does the pain get worse when you eat a meal or snack?" 3. "Have you had your amylase and lipase checked recently?" 4. "How much weight have you gained since you saw the HCP?"

2 1. A yellow haze is a sign of a toxic level of digoxin, with the client seeing through the yellow haze. Seeing a yellow haze is not the same as the client being jaundiced. In jaundice, the skin and sclera are yellow, signs of pancreatic cancer. ***2. The abdominal pain is often made worse by eating and lying supine in clients diagnosed with cancer of the pancreas.** 3. The client would not know these terms, and the HCP would be the one to check these laboratory values. 4. Clients diagnosed with cancer of the pancreas lose weight, not gain weight. TEST-TAKING HINT: The test taker could arrive at the correct answer by correlating words in the stem of the question and words in the answer options—the abdomen with eating and pain with pain.

The client diagnosed with cancer of the head of the pancreas is two (2) days postpancreatoduodenectomy (Whipple's procedure). Which nursing problem has the highest priority? 1. Anticipatory grieving. 2. Fluid volume imbalance. 3. Acute incisional pain. 4. Altered nutrition.

2 1. Clients diagnosed with cancer of the pancreas have a poor prognosis, but this is not the priority problem at this time. ***2. This is a major abdominal surgery, and there are massive fluid volume shifts that occur when this type of trauma is experienced by the body. Maintaining the circulatory system without overloading it requires extremely close monitoring.** 3. Pain is a priority but not over fluid volume status. 4. Altered nutrition would be the next highest priority. The client will be NPO with a nasogastric tube to suction and will be receiving total parenteral nutrition. TEST-TAKING HINT: The nurse should identify all of the problems, but one—fluid volume imbalance—has the greatest priority because if not addressed promptly and correctly, it could lead to severe complications.

The client diagnosed with acute pancreatitis is being discharged home. Which statement by the client indicates the teaching has been effective? 1. "I should decrease my intake of coffee, tea, and cola." 2. "I will eat a low-fat diet and avoid spicy foods." 3. "I will check my amylase and lipase levels daily." 4. "I will return to work tomorrow but take it easy."

2 1. Coffee, tea, and cola stimulate gastric and pancreatic secretions and may precipitate pain, so these foods should be avoided, not decreased. ***2. High-fat and spicy foods stimulate gastric and pancreatic secretions and may precipitate an acute pancreatic attack.** 3. Amylase and lipase levels must be checked via venipuncture with laboratory tests, and there are no daily tests the client can monitor at home. 4. The client will be fatigued as a result of decreased metabolic energy production and will need to rest and not return to work immediately. TEST-TAKING HINT: The test taker should be careful with words such as "decrease" because many times the client must avoid certain foods and situations completely, not decrease their intake of them. There are only a few blood studies that can be monitored at home on a daily basis—mainly glucose levels, which would cause the test taker to eliminate option "3."

The client has developed iatrogenic Cushing's disease. Which statement is the scientific rationale for the development of this diagnosis? 1. The client has an autoimmune problem causing the destruction of the adrenal cortex. 2. The client has been taking steroid medications for an extended period for another disease process. 3. The client has a pituitary gland tumor causing the adrenal glands to produce too much cortisol. 4. The client has developed an adrenal gland problem for which the health-care provider does not have an explanation.

2 1. Cushing's disease is not an autoimmune problem. **2. "Iatrogenic" means a problem has been caused by a medical treatment or procedure—in this case, treatment with steroids for another problem. Clients taking steroids over a period of time develop the clinical manifestations of Cushing's disease. Disease processes for which long-term steroids are prescribed include chronic obstructive pulmonary disease, cancer, and arthritis.** 3. This could be a cause for primary Cushing's syndrome. 4. There is a known reason for the client to have iatrogenic Cushing's syndrome. TEST-TAKING HINT: This question requires the test taker to know basic medical terminology.

The home health nurse is admitting a client diagnosed with cancer of the pancreas. Which information is the most important for the nurse to discuss with the client? 1. Determine the client's food preferences. 2. Ask the client if there is an advance directive. 3. Find out about insurance/Medicare reimbursement. 4. Explain that the client should eat as much as possible.

2 1. Food preferences are important for the caregiver to know because this will be the person preparing meals for the client. ***2. Cancer of the pancreas has a poor prognosis for most clients, and the nurse should determine if the client has executed an advance directive regarding their wishes.** 3. This is important because of payment issues, but it is not the highest priority. 4. Clients diagnosed with cancer frequently have anorexia, and explaining that the client should eat does not mean the client will eat. TEST-TAKING HINT: The test taker would need to know general information about the disease process to answer this question, but "2" is a good choice for many terminal diseases. Remember to read the questions carefully. The home health nurse is not arranging meals for the client.

Which electrolyte replacement should the nurse anticipate being ordered by the health-care provider in the client diagnosed with diabetic ketoacidosis (DKA) who has just been admitted to the ICU? 1. Glucose. 2. Potassium. 3. Calcium. 4. Sodium.

2 1. Glucose is elevated in DKA; therefore, the HCP would not be replacing glucose. **2. The client in DKA loses potassium from increased urinary output, acidosis, catabolic state, and vomiting. Replacement is essential for preventing cardiac dysrhythmias secondary to hypokalemia.** 3. Calcium is not affected in the client with DKA. 4. The prescribed IV for DKA—0.9% normal saline—has sodium, but it is not specifically ordered for sodium replacement. This is an isotonic solution. TEST-TAKING HINT: Option "1" should be eliminated because the problem with DKA is elevated glucose, so the HCP would not be replacing it. The test taker should use physiology knowledge and realize potassium is in the cell.

The client with type 2 diabetes controlled with biguanide oral diabetic medication is scheduled for a computed tomography (CT) scan with contrast of the abdomen to evaluate pancreatic function. Which intervention should the nurse implement? 1. Provide a high-fat diet 24 hours prior to test. 2. Hold the biguanide medication for 48 hours prior to test. 3. Obtain an informed consent form for the test. 4. Administer pancreatic enzymes prior to the test.

2 1. High-fat diets are not recommended for clients diagnosed with diabetes, and food does not have an effect on a CT scan with contrast. **2. Biguanide medication must be held for a test with contrast medium because it increases the risk of lactic acidosis, which leads to renal problems.** 3. Informed consent is not required for a CT scan. The admission consent covers routine diagnostic procedures. 4. Pancreatic enzymes are administered when the pancreas cannot produce amylase and lipase, not when the beta cells cannot produce insulin. TEST-TAKING HINT: The test taker could eliminate option "1" because high-fat diets are not recommended for any client. Because the stem specifically refers to the biguanide medication and CT contrast, a good choice addresses both of these. Option "2" discusses both the medication and the test.

The client is admitted to the intensive care department diagnosed with myxedemacoma. Which assessment data warrant immediate intervention by the nurse? 1. Serum blood glucose level of 74 mg/dL. 2. Pulse oximeter reading of 90%. 3. Telemetry reading showing sinus bradycardia. 4. The client is lethargic and sleeps all the time.

2 1. Hypoglycemia is expected in a client with myxedema; therefore, a 74-mg/dL blood glucose level is expected. **2. A pulse oximeter reading of less than 93% is significant. A 90% pulse oximeter reading indicates a PaO2 of approximately 60 on an arterial blood gas test; this is severe hypoxemia and requires immediate intervention.** 3. The client with myxedema coma is in an exaggerated hypothyroid state; a low pulse is expected in a client with hypothyroidism. 4. Lethargy is an expected symptom in a client diagnosed with myxedema; there-fore, this does not warrant immediate intervention

The nurse is admitting a client to the neurological intensive care unit who is postoperative transsphenoidal hypophysectomy. Which data warrant immediate intervention? 1. The client is alert to name but is unable to tell the nurse the location. 2. The client has an output of 2,500 mL since surgery and an intake of 1,000 mL. 3. The client's vital signs are T 97.6oF, P 88, R 20, and BP 130/80. 4. The client has a 3-cm amount of dark-red drainage on the turban dressing.

2 1. Neurological status is monitored every one (1) to two (2) hours. This client's neurological status appears intact. Clients waking up in an intensive care area may not be aware of their surroundings. **2. The output is more than double the intake in a short time. This client could be developing diabetes insipidus, a complication of trauma to the head. 3. These vital signs are within normal limits.** 4. A transsphenoidal hypophysectomy is performed by surgical access above the gum line and through the nasal passage. There is no dressing. A drip pad is taped below the nares. TEST-TAKING HINT: Two (2) of the answer options contain normal data and would not warrant immediate intervention. Option "4" does not match the type of surgery.

The client diagnosed with a pituitary tumor developed syndrome of inappropriate antidiuretic hormone (SIADH). Which interventions should the nurse implement? 1. Assess for dehydration and monitor blood glucose levels. 2. Assess for nausea and vomiting and weigh daily. 3. Monitor potassium levels and encourage fluid intake. 4. Administer vasopressin IV and conduct a fluid deprivation test.

2 1. The client has excess fluid and is not dehydrated, and blood glucose levels are not affected. **2. Early signs and symptoms are nausea and vomiting. The client has the syndrome of inappropriate secretion of antidiuretic (against allowing the body to urinate) hormone. In other words, the client is producing a hormone that will not allow the client to urinate.** 3. The client experiences dilutional hyponatremia, and the body has too much fluid already. 4. Vasopressin is the name of the antidiuretic hormone. Giving more increases the client's problem. Also, a water challenge test is performed, not a fluid deprivation test. TEST-TAKING HINT: The syndrome's name is confusing, with a double negative— "inappropriate" and "anti." It is helpful to put the situation in the test taker's own words to remember which way the fluids are being shifted in the body.

The UAP on the medical floor tells the nurse the client diagnosed with DKA wants something else to eat for lunch. Which intervention should the nurse implement? 1. Instruct the UAP to get the client additional food. 2. Notify the dietitian about the client's request. 3. Request the HCP increase the client's caloric intake. 4. Tell the UAP the client cannot have anything else.

2 1. The client is on a special diet and should not have any additional food. **2. The client will not be compliant with the diet if he or she is still hungry. Therefore, the nurse should request the dietitian talk to the client to try to adjust the meals so the client will adhere to the diet.** 3. The nurse does not need to notify the HCP for an increase in caloric intake. The appropriate referral is to the dietitian. 4. The client is on a special diet. The nurse needs to help the client maintain compliance with the medical treatment and should refer the client to the dietitian. TEST-TAKING HINT: The test taker should select the option attempting to ensure the client maintains compliance. The test taker should remember to work with members of the multidisciplinary health-care team.

The nurse is discharging a client diagnosed with diabetes insipidus. Which statement made by the client warrants further intervention? 1. "I will keep a list of my medications in my wallet and wear a Medic Alert bracelet." 2. "I should take my medication in the morning and leave it refrigerated at home." 3. "I should weigh myself every morning and record any weight gain." 4. "If I develop a tightness in my chest, I will call my health-care provider."

2 1. The client should keep a list of medication being taken and wear a Medic Alert bracelet. **2. Medication for DI is usually taken every eight (8) to 12 hours, depending on the client. The client should keep the medication close at hand.** 3. The client is at risk for fluid shifts. Weighing every morning allows the client to follow the fluid shifts. Weight gain indicates too much medication. 4. Tightness in the chest could be an indicator the medication is not being tolerated; if this occurs, the client should notify the health-care provider. TEST-TAKING HINT: This is an "except" question. This means all answers except one will be actions the client should do. If the test taker missed interpreting this from the stem, then the test taker could jump to the first action the client should do as the correct answer.

Which client problem has priority for the client diagnosed with acute pancreatitis? 1. Risk for fluid volume deficit. 2. Alteration in comfort. 3. Imbalanced nutrition: less than body requirements. 4. Knowledge deficit.

2 1. The client will be NPO to help decrease pain, but it is not the priority problem because the client will have intravenous fluids. ***2. Autodigestion of the pancreas results in severe epigastric pain, accompanied by nausea, vomiting, abdominal tenderness, and muscle guarding.** 3. Nutritional imbalance would be a possible client problem, but it is not priority. 4. Knowledge deficit is always a client problem, but it is not priority over pain. TEST-TAKING HINT: The test taker should apply Maslow's Hierarchy of Needs when selecting the priority problem for a client. After airway, pain is usually priority

The nurse is preparing to administer sliding-scale insulin to a client with type 2 diabetes. The medication administration record is as follows: ***missing a picture that shows the order for regular insulin*** At 1130, the client has a blood glucometer level of 322. Which intervention should the nurse implement? 1. Notify the health-care provider. 2. Administer 10 units of regular insulin. 3. Administer five (5) units of Humalog insulin. 4. Administer 10 units of intermediate-acting insulin.

2 1. The client's blood glucose level does not warrant notifying the HCP. **2. According to the sliding scale, any blood glucose reading between 301 and 450 requires 10 units of regular insulin, which is fast-acting insulin.** 3. Humalog is rapid-acting insulin, but the order reads regular insulin. 4. Intermediate-acting insulin, NPH or Humulin N, is not regular insulin.

The client is admitted to rule out Cushing's syndrome. Which laboratory tests should the nurse anticipate being ordered? 1. Plasma drug levels of quinidine, digoxin, and hydralazine. 2. Plasma levels of ACTH and cortisol. 3. A 24-hour urine for metanephrine and catecholamine. 4. Spot urine for creatinine and white blood cells (WBCs).

2 1. The drugs quinidine, digoxin, and hydralazine can interfere with adrenal gland secretions and cause hypofunction. Cushing's syndrome is adrenal gland hyperfunction. **2. The adrenal gland secretes cortisol and the pituitary gland secretes adrenocorticotropic hormone (ACTH), a hormone used by the body to stimulate the production of cortisol.** 3. A 24-hour urine specimen for 17-hydroxycorticosterone and 17-ketosteroid may be collected. Metanephrines and catecholamines are urine collections for pheochromocytomas. 4. Spot urinalysis and white blood cell count will not provide information on adrenal gland functions. TEST-TAKING HINT: If the test taker is aware the adrenal gland produces cortisol, then there is only one answer option which refers to cortisol.

Which signs/symptoms should make the nurse suspect the client is experiencing a thyroid storm? 1. Obstipation and hypoactive bowel sounds. 2. Hyperpyrexia and extreme tachycardia. 3. Hypotension and bradycardia. 4. Decreased respirations and hypoxia.

2 1. These are signs of myxedema (hypothyroidism) coma. Obstipation is extreme constipation. **2. Hyperpyrexia (high fever) and heart rate above 130 beats per minute are signs of thyroid storm, a severely exaggerated hyperthyroidism.** 3. Decreased blood pressure and slow heart rate are signs of myxedema coma. 4. These are signs/symptoms of myxedema coma. TEST-TAKING HINT: If the test taker does not have the knowledge to answer the question, the test taker should look at the options closely. Options "1," "3," and "4" all have signs/symptoms of "decrease"— hypoactive, hypotension, and hypoxia. The test taker should select the option which does not match.

Which assessment data indicate the client diagnosed with diabetic ketoacidosis is responding to the medical treatment? 1. The client has tented skin turgor and dry mucous membranes. 2. The client is alert and oriented to date, time, and place. 3. The client's ABG results are pH 7.29, Paco2 44, HCO3 15. 4. The client's serum potassium level is 3.3 mEq/L.

2 1. This indicates the client is dehydrated, which does not indicate the client is getting better. **2. The client's level of consciousness can be altered because of dehydration and acidosis. If the client's sensorium is intact, the client is getting better and responding to the medical treatment.** 3. These ABGs indicate metabolic acidosis; therefore, the client is not responding to treatment. 4. This potassium level is low and indicates hypokalemia, which shows the client is not responding to medical treatment. TEST-TAKING HINT: The phrase "responding to medical treatment" is asking the test taker to determine which data indicate the client is getting better. The correct answer will be normal data, and the other three (3) options will be signs/symptoms of the disease process or condition.

The nurse at a freestanding health-care clinic is caring for a 56-year-old male client who is homeless and is a type 2 diabetic controlled with insulin. Which action is an example of client advocacy? 1. Ask the client if he has somewhere he can go and live. 2. Arrange for someone to give him insulin at a local homeless shelter. 3. Notify Adult Protective Services about the client's situation. 4. Ask the HCP to take the client off insulin because he is homeless.

2 1. This is an example of interviewing the client; it is not an example of client advocacy. **2. Client advocacy focuses support on the client's autonomy. Even if the nurse disagrees with his living on the street, it is the client's right. Arranging for someone to give him his insulin provides for his needs and allows his choices.** 3. Adult Protective Services is an organization investigating any actual or potential abuse in adults. This client is not being abused by anyone. 4. The client needs the insulin to control the diabetes, and talking to the HCP about taking him off a needed medication is not an example of advocacy. TEST-TAKING HINT: Remember, the test taker must understand what the question is asking and the definition of the terms.

The client diagnosed with cancer of the pancreas is being discharged to start chemotherapy in the HCP's office. Which statement made by the client indicates the client understands the discharge instructions? 1. "I will have to see the HCP every day for six (6) weeks for my treatments." 2. "I should write down all my questions so I can ask them when I see the HCP." 3. "I am sure that this is not going to be a serious problem for me to deal with." 4. "The nurse will give me an injection in my leg and I will get to go home."

2 1. This would be the routine for radiation therapy, but chemotherapy is given one (1) to three (3) or four (4) days in a row and then a period of three (3) to four (4) weeks will elapse before the next treatment. This is called intermittent pulse therapy. ***2. The most important person in the treatment of the cancer is the client. Research has proved that the more involved a client becomes in his or her care, the better the prognosis. Clients should have a chance to ask all the questions that they have.** 3. Cancer of any kind is a serious problem. 4. Most antineoplastic medications are given intravenously. Many of the medications can cause severe complications if given intramuscularly. TEST-TAKING HINT: The test taker can eliminate option "3" on the basis that this statement is denial of the problem.

An 18-year-old female client, 5'4" tall, weighing 113 kg, comes to the clinic for a nonhealing wound on her lower leg, which she has had for two weeks. Which disease process should the nurse suspect the client has developed? 1. Type 1 diabetes. 2. Type 2 diabetes. 3. Gestational diabetes. 4. Acanthosis nigricans.

2 1. Type 1 diabetes usually occurs in young clients who are underweight. In this disease, there is no production of insulin from the beta cells in the pancreas. People with type 1 diabetes are insulin dependent with a rapid onset of symptoms, including polyuria, polydipsia, and polyphagia. **2. Type 2 diabetes is a disorder usually occurring around the age of 40, but it is now being detected in children and young adults as a result of obesity and sedentary lifestyles. Nonhealing wounds are a hallmark sign of type 2 diabetes. This client weighs 248.6 pounds and is short.** 3. Gestational diabetes occurs during pregnancy. 4. Acanthosis nigricans (AN), dark pigmentation and skin creases in the neck, is a sign of hyperinsulinemia. The pancreas is secreting excess amounts of insulin as a result of excessive caloric intake. It is identified in young children and is a precursor to the development of type 2 diabetes. TEST-TAKING HINT: The test taker must be aware of kilograms and pounds. The stem is asking about a disease process and acanthosis nigricans is a clinical manifestation of a disease, not a disease itself. Therefore, the test taker should not select this as a correct answer.

The client has had a total pancreatectomy and splenectomy for cancer of the body of the pancreas. Which discharge instructions should the nurse teach? Select all that apply. 1. Keep a careful record of intake and output. 2. Use a stool softener or bulk laxative regularly. 3. Use correct insulin injection technique. 4. Take the pain medication before the pain gets too bad. 5. Sleep with the head of the bed on blocks

2, 3, 4 1. The client is being discharged. There is no need for the client to continue recording intake and outputs at home. ***2. The client has undergone a radical and extensive surgery and will need to be administered narcotic pain medication, and a bowel regimen should be in place to prevent constipation.** ***3. Removal of the pancreas will create a diabetic state for the client. The client will need insulin and pancreatic enzyme replacement.** ***4. Client should not allow pain to reach above a "5" before taking pain medication or it will be more difficult to get the pain under control.** 5. There is no reason for the client to sleep with the head of the bed elevated. TEST-TAKING HINT: The test taker might choose option "3" by remembering that the pancreas secretes insulin. Option "4" is taught to all clients in pain.

The emergency department nurse is caring for a client diagnosed with HHNS who has a blood glucose of 680 mg/dL. Which question should the nurse ask the client to determine the cause of this acute complication? 1. "When is the last time you took your insulin?" 2. "When did you have your last meal?" 3. "Have you had some type of infection lately?" 4. "How long have you had diabetes?"

3 1. A client with type 2 diabetes usually is prescribed oral hypoglycemic medications, not insulin. 2. The client could not eat enough food to cause a 680-mg/dL blood glucose level; therefore, this question does not need to be asked. **3. The most common precipitating factor is infection. The manifestations may be slow to appear, with onset ranging from 24 hours to two (2) weeks.** 4. This does not help determine the cause of this client's HHNS. TEST-TAKING HINT: If the test taker does not know the answer to this question, the test taker could possibly relate to the phrase "acute complication" and realize a medical problem might cause this and select infection, option "3."

The nurse is admitting a client diagnosed with primary adrenal cortex insufficiency (Addison's disease). Which clinical manifestations should the nurse expect to assess? 1. Moon face, buffalo hump, and hyperglycemia. 2. Hirsutism, fever, and irritability. 3. Bronze pigmentation, hypotension, and anorexia. 4. Tachycardia, bulging eyes, and goiter.

3 1. A moon face, buffalo hump, and hyperglycemia result from Cushing's syndrome, hyperfunction of the adrenal gland. 2. Hirsutism is hair growth where it normally does not occur, such as facial hair on women. Fever and irritability, along with hirsutism, are clinical manifestations of Cushing's syndrome. 3. Bronze pigmentation of the skin, particularly of the knuckles and other areas of skin creases, occurs in Addison's disease. Hypotension and anorexia also occur with Addison's disease. 4. Tachycardia, bulging eyes, and goiter are clinical manifestations occurring with thyroid disorders. TEST-TAKING HINT: This question contains answer options referring to opposite diseases, Addison's disease and Cushing's syndrome. If two options—in this case, options "1" and "2"—are appropriate for one of the diseases, then these two can be ruled out as the correct answer.

The nurse is teaching the client diagnosed with type 2 diabetes mellitus about diet. Which diet selection indicates the client understands the teaching? 1. A submarine sandwich, potato chips, and diet cola. 2. Four (4) slices of a supreme thin-crust pizza and milk. 3. Smoked turkey sandwich, celery sticks, and unsweetened tea. 4. A roast beef sandwich, fried onion rings, and a cola.

3 1. A submarine sandwich is on a bun-type bread and is usually six (6) to 12 inches long, and potato chips add fat and more carbohydrates to the meal. 2. Four (4) slices of pizza contain excessive numbers of carbohydrates, plus cheese and meats, and whole milk is high in fat. **3. Turkey is a low-fat meat. A sandwich usually means normal slices of bread, and the client needs at least 50% carbohydrates in each meal. Celery sticks are not counted as carbohydrates.** 4. The roast beef sandwich is high in carbohydrates, fried onion rings are high in fat, and a regular cola is high in carbohydrates.

60. The client diagnosed with Cushing's disease has undergone a unilateral adrenalectomy. Which discharge instructions should the nurse discuss with the client? 1. Instruct the client to take the glucocorticoid and mineralocorticoid medications as prescribed. 2. Teach the client regarding sexual functioning and androgen replacement therapy. 3. Explain the signs and symptoms of infection and when to call the health-care provider. 4. Demonstrate turn, cough, and deep-breathing exercises the client should perform every two (2) hours.

3 1. A unilateral adrenalectomy results in one adrenal gland still functioning. No hormone replacement will be required. 2. The client can still have normal physiological functioning, including sexual functioning, with the remaining gland. **3. Notifying the HCP if signs/symptoms of infection develop is an instruction given to all surgical clients on discharge.** 4. Turning and coughing is taught prior to surgery, not at discharge. TEST-TAKING HINT: The test taker must notice the adjectives; "discharge" tells the reader a time frame for the instructions. This rules out option "4."

The nurse is completing discharge teaching to the client diagnosed with acute pancreatitis. Which instruction should the nurse discuss with the client? 1. Instruct the client to decrease alcohol intake. 2. Explain the need to avoid all stress. 3. Discuss the importance of stopping smoking. 4. Teach the correct way to take pancreatic enzymes.

3 1. Alcohol must be avoided entirely because it can cause stones to form, blocking pancreatic ducts and the outflow of pancreatic juice, causing further inflammation and destruction of the pancreas. 2. Stress stimulates the pancreas and should be dealt with, but it is unrealistic to think that a client can avoid all stress. By definition, the absence of all stress is death. ***3. Smoking stimulates the pancreas to release pancreatic enzymes and should be stopped.** 4. The client has acute pancreatitis, and pancreatic enzymes are only needed for chronic pancreatitis. TEST-TAKING HINT: The test taker should eliminate "2" because of the word "all," which is an absolute and there are very few absolutes in health care. The test taker should note the adjective "acute" in the stem, which may help the test taker eliminate option "4" because enzymes are given for a chronic condition.

The male client diagnosed with chronic pancreatitis calls and reports to the clinic nurse that he has been having a lot of "gas," along with frothy and very foul-smelling stools. Which action should the nurse take? 1. Explain that this is common for chronic pancreatitis. 2. Ask the client to bring in a stool specimen to the clinic. 3. Arrange an appointment with the HCP for today. 4. Discuss the need to decrease fat in the diet so that this won't happen.

3 1. Any change in the client's stool should be a cause for concern to the clinic nurse. 2. This is not necessary because the nurse knows changes in stool occur as a complication of pancreatitis and the client needs to see the HCP. ***3. Steatorrhea (fatty, frothy, foul-smelling stool) is caused by a decrease in pancreatic enzyme secretion and indicates impaired digestion and possibly an increase in the severity of the pancreatitis. The client should see the HCP.** 4. Decreasing fat in the diet will not help stop this type of stool. TEST-TAKING HINT: This question requires the test taker to have knowledge of the disease process, but if the test taker knows that the exocrine function of the pancreas is part of the gastrointestinal system, the test taker might think that altered stool would be cause for concern.

The nurse writes a problem of "altered body image" for a 34-year-old client diagnosed with Cushing's disease. Which intervention should be implemented? 1. Monitor blood glucose levels prior to meals and at bedtime. 2. Perform a head-to-toe assessment on the client every shift. 3. Use therapeutic communication to allow the client to discuss feelings. 4. Assess bowel sounds and temperature every four (4) hours.

3 1. Blood glucose levels do not address the problem of altered body image. 2. Head-to-toe assessments are performed to detect a physiological problem, not a psychosocial one. **3. Allowing the client to ventilate feelings about the altered body image is the most appropriate intervention. The nurse cannot do anything to help the client's buffalo hump or moon face.** 4. Bowel sounds and temperature are physical symptoms. TEST-TAKING HINT: The intervention must match the problem.

The nurse is caring for clients on a medical floor. Which client should be assessed first? 1. The client diagnosed with syndrome of inappropriate antidiuretic hormone (SIADH) who has a weight gain of 1.5 pounds since yesterday. 2. The client diagnosed with a pituitary tumor who has developed diabetes insipidus (DI) and has an intake of 1,500 mL and an output of 1,600 mL in the last 8 hours. 3. The client diagnosed with syndrome of inappropriate antidiuretic hormone (SIADH) who is having muscle twitching. 4. The client diagnosed with diabetes insipidus (DI) who is complaining of feeling tired after having to get up at night.

3 1. Clients with SIADH have a problem with retaining fluid. This is expected. 2. This client's intake and output are relatively the same. 3. Muscle twitching is a sign of early sodium imbalance. If an immediate intervention is not made, the client could begin to seize. 4. The client has to get up all night to urinate, so the client feeling tired is expected. TEST-TAKING HINT: All of the answer options contain expected information except option "3."

The client admitted to rule out pancreatic islet tumors complains of feeling weak, shaky, and sweaty. Which should be the first intervention implemented by the nurse? 1. Start an IV with D5W. 2. Notify the health-care provider. 3. Perform a bedside glucose check. 4. Give the client some orange juice.

3 1. The client may need IV medication, but in this case if it is needed, it would be 50% dextrose. 2. The HCP might be notified, but the nurse needs to assess the client first. ***3. These are symptoms of an insulin reaction (hypoglycemia). A bedside glucose check should be done. Pancreatic islet tumors can produce hyperinsulinemia or hypoglycemia.** 4. This would be done after the nurse knows the glucose reading. TEST-TAKING HINT: The test taker should remember the function of the pancreas. This would lead the test taker

The nurse is assessing the feet of a client with long-term type 2 diabetes. Which assessment data warrant immediate intervention by the nurse? 1. The client has crumbling toenails. 2. The client has athlete's foot. 3. The client has a necrotic big toe. 4. The client has thickened toenails.

3 1. Crumbling toenails indicate tinea unguium, which is a fungus infection of the toenail. 2. Athlete's foot is a non-life-threatening fungal infection. **3. A necrotic big toe indicates "dead" tissue. The client does not feel pain, does not realize the injury, and does not seek treatment. Increased blood glucose levels decrease the oxygen supply needed to heal the wound and increase the risk for developing an infection.** 4. Big, thick toenails are fungal infections and do not require immediate intervention by the nurse. TEST-TAKING HINT: The test taker should select the option indicating this is possibly a life-threatening complication or some type of assessment data the health-care provider should be informed of immediately. Remember "warrants immediate intervention."

The male client diagnosed with syndrome of inappropriate antidiuretic hormone (SIADH) secondary to cancer of the lung tells the nurse he wants to discontinue the fluid restriction and does not care if he dies. Which action by thenurse is an example of the ethical principle of autonomy? 1. Discuss the information the client told the nurse with the health-care provider and significant other. 2. Explain it is possible the client could have a seizure if he drank fluid beyond the restrictions. 3. Notify the health-care provider of the client's wishes and give the client fluids as desired. 4. Allow the client an extra drink of water and explain the nurse could get into trouble if the client tells the health-care provider.

3 1. Discussing the information with others is not allowing the client to decide what is best for himself. 2. This could be an example of beneficence (to do good) if the nurse did this so the client has information on which to base a decision on whether to continue the fluid restriction. **3. This is an example of autonomy (the client has the right to decide for himself).** 4. This is an example of dishonesty and should never be tolerated in a health-care setting.

The client has just had an endoscopic retrograde cholangiopancreatogram (ERCP). Which post-procedure intervention should the nurse implement? 1. Assess for rectal bleeding. 2. Increase fluid intake. 3. Assess gag reflex. 4. Keep in supine position.

3 1. During this procedure a scope is placed down the client's mouth; therefore, assessing for rectal bleeding would not be a common intervention. 2. The client's throat has been anesthetized to insert the scope; therefore, fluid and food are withheld until the gag reflex has returned. ***3. The gag reflex will be suppressed as a result of the local anesthesia applied to the throat to insert the endoscope into the esophagus; therefore, the gag reflex must be assessed prior to allowing the client to resume eating or drinking.** 4. The client should be in a semi-Fowler's or side-lying position to prevent aspiration. TEST-TAKING HINT: The test taker should apply the nursing process and select an option that addresses assessment—either "1" or "3." The medical suffix "endo" should help the test taker select "3" as the correct answer.

The client diagnosed with type 2 diabetes is admitted to the intensive care unit (ICU) with hyperosmolar hyperglycemic nonketotic syndrome (HHNS) coma. Which assessment data should the nurse expect the client to exhibit? 1. Kussmaul's respirations. 2. Diarrhea and epigastric pain. 3. Dry mucous membranes. 4. Ketone breath odor.

3 1. Kussmaul's respirations occur with diabetic ketoacidosis (DKA) as a result of the breakdown of fat, resulting in ketones. 2. Diarrhea and epigastric pain are not associated with HHNS. **3. Dry mucous membranes are a result of the hyperglycemia and occur with both HHNS and DKA.** 4. This occurs with DKA as a result of the breakdown of fat, resulting in ketones. TEST-TAKING HINT: The test taker must be able to differentiate between HHNS (type 2 diabetes) and DKA (type 1 diabetes), which primarily is the result of the breakdown of fat and results in an increase in ketones causing a decrease in pH, resulting in metabolic acidosis.

The client diagnosed with acute pancreatitis is in pain. Which position should the nurse assist the client to assume to help decrease the pain? 1. Recommend lying in the prone position with legs extended. 2. Maintain a tripod position over the bedside table. 3. Place in side-lying position with knees flexed. 4. Encourage a supine position with a pillow under the knees.

3 1. Lying on the stomach will not help to decrease the client's pain. 2. This is a position used by clients with chronic obstructive pulmonary disease to help lung expansion. ***3. This fetal position decreases pain caused by stretching of the peritoneum as a result of edema.** 4. Laying supine causes the peritoneum to stretch, which increases the pain. TEST-TAKING HINT: The test taker should think about where the pancreas is located in the abdomen to help identify the correct answer. Prone or supine would cause the abdomen to be stretched, which would increase pain.

The nurse is planning a program for clients at a health fair regarding the prevention and early detection of cancer of the pancreas. Which self-care activity should the nurse teach that is an example of primary nursing care? 1. Monitor for elevated blood glucose at random intervals. 2. Inspect the skin and sclera of the eyes for a yellow tint. 3. Limit meat in the diet and eat a diet that is low in fats. 4. Instruct the client with hyperglycemia about insulin injections.

3 1. Monitoring the blood glucose at random intervals, as would be done at a health fair, can pick up possible diabetes mellitus or the presence of a pancreatic tumor, but detecting a disease at an early stage is secondary screening, not primary prevention. 2. Inspecting the skin for jaundice would be a secondary nursing intervention. ***3. Limiting the intake of meat and fats in the diet would be an example of primary interventions. Risk factors for the development of cancer of the pancreas are cigarette smoking and eating a high-fat diet that is high in animal protein. By changing these behaviors the client could possibly prevent the development of cancer of the pancreas. Other risk factors include genetic predisposition and exposure to industrial chemicals.** 4. Instructing a client with hyperglycemia (diabetes mellitus) is an example of tertiary nursing care. TEST-TAKING HINT: Even if the test taker was not sure of the definition of primary, secondary, or tertiary nursing care, primary means first. Only one answer option is preventive, and preventing something comes before treating it.

The nurse is discussing ways to prevent diabetic ketoacidosis with the client diagnosed with type 1 diabetes. Which instruction is most important to discuss with the client? 1. Refer the client to the American Diabetes Association. 2. Do not take any over-the-counter (OTC) medications. 3. Take the prescribed insulin even when unable to eat because of illness. 4. Explain the need to get the annual flu and pneumonia vaccines.

3 1. The American Diabetes Association is an excellent referral, but the nurse should discuss specific ways to prevent DKA. 2. The client should be careful with OTC medications, but this intervention does not help prevent the development of DKA. **3. Illness increases blood glucose levels; therefore, the client must take insulin and consume high-carbohydrate foods such as regular Jell-O, regular popsicles, and orange juice.** 4. Vaccines are important to help prevent illness, but regardless of whether the client gets these vaccines, the client can still develop diabetic ketoacidosis. TEST-TAKING HINT: The words "most important" in the stem of the question indicate one (1) or more options may be appropriate instructions but only one (1) is the priority intervention.

The client diagnosed with diabetes complains of a curtain being drawn across the eyes. Which should the nurse implement first? 1. Assess the eyes using an ophthalmoscope. 2. Tell the client to keep the eyes closed. 3. Notify the health-care provider (HCP). 4. Call the Rapid Response Team (RRT).

3 1. The HCP and not the nurse should perform this assessment. The nurse has an unusual and potentially life-changing issue identified. 2. Keeping the eyes closed will not change the outcome of retinal detachment. This is an ophthalmological emergency. **3. This is an emergency; this indicates retinal detachment. The nurse should notify the HCP.** 4. The RRT will help to prevent a cardiac or respiratory arrest. The HCP should be notified to arrange for an ophthalmologist consult. TEST-TAKING HINT: The test taker should recognize life-changing or life-threatening complications of a disease process. Failure to immediately intervene can result in a "failure to rescue" situation.

The client diagnosed with type 1 diabetes has a glycosylated hemoglobin (A1c) of 8.1%. Which interpretation should the nurse make based on this result? 1. This result is below normal levels. 2. This result is within acceptable levels. 3. This result is above recommended levels. 4. This result is dangerously high.`

3 1. The acceptable level for an A1c for a client with diabetes is between 6% and 7%, which corresponds to a 120- to 140-mg/dL average blood glucose level. 2. This result is not within acceptable levels for the client with diabetes, which is 6% to 7%. **3. This result parallels a serum blood glucose level of approximately 180 to 200 mg/dL. An A1c is a blood test reflecting average blood glucose levels over a period of three (3) months; clients with elevated blood glucose levels are at risk for developing long-term complications.** 4. An A1c of 13% is dangerously high; it reflects a 300-mg/dL average blood glucose level over the past three (3) months. TEST-TAKING HINT: The test taker must know normal and abnormal diagnostic laboratory values. Laboratory values vary depending on which laboratory performs the test.

The client received 10 units of Humulin R, a fast-acting insulin, at 0700. At 1030 the unlicensed assistive personnel (UAP) tells the nurse the client has a headache and is really acting "funny." Which intervention should the nurse implement first? 1. Instruct the UAP to obtain the blood glucose level. 2. Have the client drink eight (8) ounces of orange juice. 3. Go to the client's room and assess the client for hypoglycemia. 4. Prepare to administer one (1) ampule 50% dextrose intravenously.

3 1. The blood glucose level should be obtained, but it is not the first intervention. 2. If it is determined the client is having a hypoglycemic reaction, orange juice is appropriate. **3. Regular insulin peaks in two (2) to four (4) hours. Therefore, the nurse should think about the possibility the client is having a hypoglycemic reaction and should assess the client. The nurse should not delegate nursing tasks to a UAP if the client is unstable.** 4. Dextrose 50% is only administered if the client is unconscious and the nurse suspects hypoglycemia. TEST-TAKING HINT: When answering a question requiring the nurse to decide which intervention to implement first, all four options are plausible for the situation but only one answer should be implemented first. The test taker must apply the nursing process; assessment is the first step of the nursing process.

The nurse is discussing complications of chronic pancreatitis with a client diagnosed with the disease. Which complication should the nurse discuss with the client? 1. Diabetes insipidus. 2. Crohn's disease. 3. Narcotic addiction. 4. Peritonitis.

3 1. The client is at risk for diabetes mellitus (destruction of beta cells), not diabetes insipidus, a disorder of the pituitary gland. 2. Crohn's disease is an inflammatory disorder of the lining of the gastrointestinal system, especially of the terminal ileum. ***3. Narcotic addiction is related to the frequent, severe pain episodes that often occur with chronic pancreatitis and its complications and that require narcotics for relief.** 4. Peritonitis, an inflammation of the lining of the abdomen, is not a common complication of chronic pancreatitis. TEST-TAKING HINT: The test taker may be able to delete options based on normal anatomical and physiological data. Diabetes insipidus is a complication of the pituitary gland; Crohn's disease is a disease of the GI tract; and the peritoneum is the lining of the abdomen. Therefore, options "1," "2," and "4" can be eliminated.

The client is admitted to the medical unit with a diagnosis of rule-out diabetes insipidus (DI). Which instructions should the nurse teach regarding a fluid deprivation test? 1. The client will be asked to drink 100 mL of fluid as rapidly as possible and then will not be allowed fluid for 24 hours. 2. The client will be administered an injection of antidiuretic hormone (ADH), and urine output will be measured for four (4) to six (6) hours. 3. The client will have nothing by mouth (NPO), and vital signs and weights will be done hourly until the end of the test. 4. An IV will be started with normal saline, and the client will be asked to try to hold the urine in the bladder until a sonogram can be done.

3 1. The client is not allowed to drink during the test. 2. This test does not require any medications to be administered, and vasopressin will treat the DI, not help diagnose it. **3. The client is deprived of all fluids, and if the client has DI the urine production will not diminish. Vital signs and weights are taken every hour to determine circulatory status. If a marked decrease in weight or vital signs occurs, the test is immediately terminated.** 4. No fluid is allowed and a sonogram is not involved. TEST-TAKING HINT: The name of the test is a fluid deprivation test. Two (2) of the options require the administration of some type of fluid.

The unlicensed assistive personnel (UAP) complains to the nurse she has filled the water pitcher four (4) times during the shift for a client diagnosed with a closed head injury and the client has asked for the pitcher to be filled again. Which intervention should the nurse implement first? 1. Tell the UAP to fill the pitcher with ice cold water. 2. Instruct the UAP to start measuring the client's I&O. 3. Assess the client for polyuria and polydipsia. 4. Check the client's BUN and creatinine levels.

3 1. The client should have the water pitcher filled, but this is not the first action. 2. This should be done but not before assessing the problem. **3. The first action should be to determine if the client is experiencing polyuria and polydipsia as a result of developing diabetes insipidus, a complication of the head trauma.** 4. This could be done, but it will not give the nurse information about DI. TEST-TAKING HINT: The nurse must apply a systematic approach to answering priority questions. Maslow's hierarchy of needs should be applied if it is a physiological problem and the nursing process if it is a question of this nature. Assessment is the first step in the nursing process.

The client is being admitted to the outpatient department prior to an endoscopic retrograde cholangiopancreatogram (ERCP) to rule out cancer of the pancreas. Which pre-procedure instruction should the nurse teach? 1. Prepare to be admitted to the hospital after the procedure for observation. 2. If something happens during the procedure, then emergency surgery will be done. 3. Do not eat or drink anything after midnight the night before the test. 4. If done correctly, this procedure will correct the blockage of the stomach.

3 1. The client should stay in the outpatient department after the procedure for observation unless the HCP determines that a more extensive workup should be completed. 2. This is not the type of procedure where the results warrant an emergency surgery. A cardiac catheterization sometimes results in an emergency surgery and the client is prepared for this possibility, but this is not the case with an ERCP. ***3. The client should be NPO after midnight to make sure the stomach is empty to reduce the risk of aspiration during the procedure.** 4. The possible blockage would be of the duodenum, common bile duct, or pancreatic outlet. TEST-TAKING HINT: The nurse should never preface any instruction with "if done correctly" because this sets the nurse, HCP, and facility up for a lawsuit. The client is NPO for any procedure or surgery where the client will receive general or twilight sleep anesthesia.

The client diagnosed with hypothyroidism is prescribed the thyroid hormone levothyroxine (Synthroid). Which assessment data indicate the medication has been effective? 1. The client has a three (3)-pound weight gain. 2. The client has a decreased pulse rate. 3. The client's temperature is WNL. 4. The client denies any diaphoresis.

3 1. The medication will help increase the client's metabolic rate. A weight gain indicates not enough medication is being taken to put the client in a euthyroid (normal thyroid) state. 2. A decreased pulse rate indicates there is not enough thyroid hormone level; therefore, the medication is not effective. **3. The client with hypothyroidism frequently has a subnormal temperature,so a temperature WNL indicates the medication is effective.** 4. Diaphoresis (sweating) occurs with hyper-thyroidism, not hypothyroidism

The nurse is preparing to administer the following medications. Which medication should the nurse question administering? 1. The thyroid hormone to the client who does not have a T3, T4 level. 2. The regular insulin to the client with a blood glucose level of 210 mg/dL. 3. The loop diuretic to the client with a potassium level of 3.3 mEq/L. 4. The cardiac glycoside to the client who has a digoxin level of 1.4 mg/dL.

3 1. The thyroid hormone must be administered daily, and thyroid levels are drawn every six (6) months or so. 2. A blood glucose level of 210 mg/dL requires insulin administration; therefore,the nurse should not question administering this medication **3. This potassium level is below normal,which is 3.5 to 5.5 mEq/L. Therefore,the nurse should question administering this medication because loop diuretics cause potassium loss in the urine.** 4. The digoxin level is within therapeutic range—0.8 to 2.0 mg/dL; therefore, the nurse should administer this medication.

The client is admitted to the medical department with a diagnosis of R/O acute pancreatitis. Which laboratory value should the nurse monitor to confirm this diagnosis? 1. Creatinine and BUN. 2. Troponin and CPK-MB. 3. Serum amylase and lipase. 4. Serum bilirubin and calcium.

3 1. These laboratory values are monitored for clients in kidney failure. 2. These laboratory values are elevated in clients with a myocardial infarction. **3. Serum amylase increases within 2 to 12 hours of the onset of acute pancreatitis to 2 to 3 times normal and returns to normal in 3 to 4 days; lipase elevates and remains elevated for 7 to 14 days.** 4. Bilirubin may be elevated as a result of compression of common duct, and hypocalcemia develops in up to 25% of clients with acute pancreatitis, but these laboratory values do not confirm the diagnosis. TEST-TAKING HINT: The test taker must be able to identify at least two (2) laboratory values that reflect each organ function prior to taking the RN-NCLEX. There is really no Test- Taking Hint that can help select the correct answer; this is knowledge.

The charge nurse is making client assignments in the intensive care unit. Which client should be assigned to the most experienced nurse? 1. The client with type 2 diabetes who has a blood glucose level of 348 mg/dL. 2. The client diagnosed with type 1 diabetes who is experiencing hypoglycemia. 3. The client with DKA who has multifocal premature ventricular contractions. 4. The client with HHNS who has a plasma osmolarity of 290 mOsm/L.

3 1. This blood glucose level is elevated, but not life threatening, in the client diagnosed with type 2 diabetes. Therefore, a less experienced nurse could care for this client. 2. Hypoglycemia is an acute complication of type 1 diabetes, but it can be managed by frequent monitoring, so a less experienced nurse could care for this client. **3. Multifocal PVCs, which are secondary to hypokalemia and can occur in clients with DKA, are a potentially life-threatening emergency. This client needs an experienced nurse.** 4. A plasma osmolarity of 280 to 300 mOsm/L is within normal limits; therefore, a less experienced nurse could care for this client. TEST-TAKING HINT: The test taker must select the client with an abnormal, an unexpected, or a life-threatening sign/symptom for his or her disease process and assign this client to the most experienced nurse.

The charge nurse of an intensive care unit is making assignments for the night shift. Which client should be assigned to the most experienced intensive care nurse? 1. The client diagnosed with respiratory failure who is on a ventilator and requires frequent sedation. 2. The client diagnosed with lung cancer and iatrogenic Cushing's disease with ABGs of pH 7.35, Pao2 88, Paco2 44, and HCO3 22. 3. The client diagnosed with Addison's disease who is lethargic and has a BP of 80/45, P 124, and R 28. 4. The client diagnosed with hyperthyroidism who has undergone a thyroidectomy two (2) days ago and has a negative Trousseau's sign.

3 1. This client could be cared for by any nurse qualified to work in an intensive care unit. 2. These blood gases are within normal limits. **3. This client has a low blood pressure and tachycardia. This client may be experiencing an addisonian crisis, a potentially life-threatening condition. The most experienced nurse should** care for this client. 4. A negative Trousseau's sign is normal for this client. TEST-TAKING HINT: The answer options "1," "2," and "4" have expected or normal data. Only one option has abnormal data. Even if the test taker is unaware of addisonian crisis, these are vital signs indicating potential shock.

Which medication order should the nurse question in the client diagnosed with untreated hypothyroidism? 1. Thyroid hormones. 2. Oxygen. 3. Sedatives. 4. Laxatives.

3 1. Thyroid hormones are the treatment of choice for the client diagnosed with hypothyroidism; therefore, the nurse should not question this medication. 2. In untreated hypothyroidism, the medical management is aimed at supporting vital functions, so administering oxygen is an appropriate medication. **3. Untreated hypothyroidism is characterized by an increased susceptibility to the effects of most hypnotic and sedative agents; therefore, the nurse should question this medication.** 4. Clients with hypothyroidism become constipated as a result of decreased metabolism, so laxatives should not be questioned by the nurse.

The client diagnosed with type 1 diabetes is receiving Humalog, a rapid-acting insulin, by sliding scale. The order reads blood glucose level: < 150, zero (0) units; 151 to 200, three (3) units; 201 to 250, six (6) units; >251, contact health-care provider. The unlicensed assistive personnel (UAP) reports to the nurse the client's glucometer reading is 189. How much insulin should the nurse administer to the client? ______

3 units The client's result is 189, which is between 151 and 200, so the nurse should administer three (3) units of Humalog insulin subcutaneously. TEST-TAKING HINT: The test taker must be aware of the way HCPs write medication orders. HCPs order insulin on a sliding scale according to a range of blood glucose levels.

The client is diagnosed with acute pancreatitis. Which health-care provider's admitting order should the nurse question? 1. Bed rest with bathroom privileges. 2. Initiate IV therapy at D5W 125 mL/hr. 3. Weigh client daily. 4. Low-fat, low-carbohydrate diet.

4 1. Bed rest will decrease metabolic rate, gastrointestinal secretion, pancreatic secretions, and pain; therefore this HCP's order should not be questioned. 2. The client will be NPO; therefore, initiating IV therapy would be an appropriate order. 3. Short-term weight gain changes reflect fluid balance because the client will be NPO and receiving IV fluids. Daily weighing would be an appropriate HCP's order. ***4. The client will be NPO, which will decrease stimulation of the pancreatic enzymes, which will result in decreased autodigestion of the pancreas, therefore decreasing pain.** TEST-TAKING HINT: The test taker must determine which HCP's order is not expected for the diagnosis. Sometimes if the test taker asks which order would be expected, it is easier to identify the unexpected or abnormal HCP order.

Which statement made by the client makes the nurse suspect the client is experiencing hyperthyroidism? 1. "I just don't seem to have any appetite anymore." 2. "I have a bowel movement about every 3 to 4 days." 3. "My skin is really becoming dry and coarse." 4. "I have noticed all my collars are getting tighter."

4 1. Decreased appetite is a symptom of hypothyroidism, not hyperthyroidism. 2. Constipation is a symptom of hypothyroidism. 3. Dry, coarse skin is a sign of hypothyroidism **4. The thyroid gland (in the neck) en-larges as a result of the increased need for thyroid hormone production; an enlarged gland is called a goiter.**

The nurse is caring for a client diagnosed with diabetes insipidus (DI). Which intervention should be implemented? 1. Administer sliding-scale insulin as ordered. 2. Restrict caffeinated beverages. 3. Check urine ketones if blood glucose is >250. 4. Assess tissue turgor every four (4) hours.

4 1. Diabetes insipidus is not diabetes mellitus; sliding-scale insulin is not administered to the client. 2. There is no caffeine restriction for DI. 3. Checking urine ketones is not indicated. **4. The client is excreting large amounts of dilute urine. If the client is unable to drink enough fluids, the client will quickly become dehydrated, so tissue turgor should be assessed frequently.** TEST-TAKING HINT: Two (2) of the answer options are appropriate for diabetes mellitus, not diabetes insipidus, and can be eliminated on this basis alone.

Which client would the nurse identify as being at risk for developing diabetes? 1. The client who eats mostly candy and potatoes. 2. The 22-year-old client who has been taking birth control pills. 3. The client who has a cousin who has had diabetes for two (2) years. 4. The 38-year-old female who delivered a 10-pound infant.

4 1. Eating sweets and high-carbohydrate foods can lead to obesity, but eating candy does not cause diabetes. 2. Birth control pills do not increase the risk of developing diabetes. 3. Type 2 diabetes can be more prevalent in families, but one cousin does not increase the risk of diabetes for the client. **4. Research shows that women who delivered a large infant have a greater risk for developing diabetes.** TEST-TAKING HINT: The test taker must know antecedents of developing disease processes.

Which nursing intervention should be included in the plan of care for the client diagnosed with hyperthyroidism? 1. Increase the amount of fiber in the diet. 2. Encourage a low-calorie, low-protein diet. 3. Decrease the client's fluid intake to 1,000 mL/day. 4. Provide six (6) small, well-balanced meals a day.

4 1. Fiber should be increased in the client diagnosed with hypothyroidism because the client experiences constipation secondary to decreased metabolism. 2. The client with hyperthyroidism should have a high-calorie, high-protein diet. 3. The client's fluid intake should be increased to replace fluids lost through diarrhea and excessive sweating. **4. The client with hyperthyroidism has an increased appetite; therefore, well-balanced meals served several times throughout the day will help with the client's constant hunger.**

The nurse is discussing the importance of exercising with a client diagnosed with type 2 diabetes whose diabetes is well controlled with diet and exercise. Which information should the nurse include in the teaching about diabetes? 1. Eat a simple carbohydrate snack before exercising. 2. Carry peanut butter crackers when exercising. 3. Encourage the client to walk 20 minutes three (3) times a week. 4. Perform warm-up and cool-down exercises.

4 1. The client diagnosed with type 2 diabetes who is not taking insulin or oral agents does not need extra food before exercise. 2. The client with diabetes who is at risk for hypoglycemia when exercising should carry a simple carbohydrate, but this client is not at risk for hypoglycemia. 3. Clients with diabetes controlled by diet and exercise must exercise daily at the same time and in the same amount to control the glucose level. **4. All clients who exercise should perform warm-up and cool-down exercises to help prevent muscle strain and injury.** TEST-TAKING HINT: Options "1" and "2" apply directly to clients diagnosed with diabetes, and options "3" and "4" do not directly address clients diagnosed with diabetes. The reader could narrow the choices by either eliminating or including the two similar options.

The nurse caring for a client diagnosed with cancer of the pancreas writes the nursing diagnosis of "risk for altered skin integrity related to pruritus." Which interventions should the nurse implement? 1. Assess tissue turgor. 2. Apply antifungal creams. 3. Monitor bony prominences for breakdown. 4. Have the client keep the fingernails short.

4 1. The client is at risk for poor nutrition and malabsorption syndrome for which tissue turgor assessment is appropriate, but the client problem here is pruritus, or itching. 2. The itching is associated with the cancer and not a fungus. 3. The client should be monitored for skin breakdown, but pruritus is itching and an intervention is needed to prevent skin problems from scratching. ***4. Keeping the fingernails short will reduce the chance of breaks in the skin from scratching.** TEST-TAKING HINT: The problem is "risk for skin breakdown." The etiology is "pruritus." Interventions address the etiology. Goals address the problem

The nurse is providing an in-service on thyroid disorders. One of the attendees asks the nurse, "Why don't the people in the United States get goiters as often?" Which statement by the nurse is the best response? 1. "It is because of the screening techniques used in the United States." 2. "It is a genetic predisposition rare in North Americans." 3. "The medications available in the United States decrease goiters." 4. "Iodized salt helps prevent the development of goiters in the United States."

4 1. There is no screening for thyroid disorders, just serum thyroid levels. 2. This is not a true statement. 3. Medications do not decrease the development of goiters. **4. Almost all of the iodine entering the body is retained in the thyroid gland. A deficiency in iodine will cause the thyroid gland to work hard and enlarge, which is called a goiter. Goiters are commonly seen in geographical regions having an iodine deficiency. Most table salt in the United States has iodine added.**

Which laboratory data would the nurse identify when discussing a client with uncontrolled diabetes mellitus type 2? 1. Glucose 89 Hemoglobin A1C 5.6 2. Glucose 134 Hemoglobin A1C 3. Glucose 112 Hemoglobin A1C 8.2 4. Glucose 439 Hemoglobin A1C 9.3

4 1. These results are both WNL; they do not indicate type 2 diabetes. 2. These results are both slightly above normal but they do not indicate type 2 diabetes. 3. These results are both slightly above normal but they do not indicate type 2 diabetes. **4. Both laboratory values are above the normal range. The A1c indicates a lengthy time (at least two [2] to three [3] months) that the blood glucose has been high. This would be supportive of uncontrolled type 2 diabetes.** TEST-TAKING HINT: The test taker must know normal and abnormal diagnostic laboratory values. Laboratory values vary depending on which laboratory performs the test.

Which arterial blood gas results should the nurse expect in the client diagnosed with diabetic ketoacidosis? 1. pH 7.34, Pao2 99, Paco2 48, HCO3 24. 2. pH 7.38, Pao2 95, Paco2 40, HCO3 22. 3. pH 7.46, Pao2 85, Paco2 30, HCO3 26. 4. pH 7.30, Pao2 90, Paco2 30, HCO3 18.

4 1. This ABG indicates respiratory acidosis, which is not expected. 2. This ABG is normal, which is not expected. 3. This ABG indicates respiratory alkalosis, which is not expected. **4. This ABG indicates metabolic acidosis, which is expected in a client diagnosed with diabetic ketoacidosis.** TEST-TAKING HINT: The test taker must know normal ABGs to be able to correctly answer this question. Normal ABGs are pH 7.35 to 7.45; Pao2 80 to 100; Paco2 35 to 45; HCO3 22 to 26.

The nurse is teaching a community class to people with type 2 diabetes mellitus. Which explanation explains the development of type 2 diabetes? 1. The islet cells in the pancreas stop producing insulin. 2. The client eats too many foods high in sugar. 3. The pituitary gland does not produce vasopressin. 4. The cells become resistant to the circulating insulin.

4 1. This is the cause of type 1 diabetes mellitus. 2. This may be a reason for obesity, which may lead to type 2 diabetes, but eating too much sugar does not cause diabetes. 3. This is the explanation for diabetes insipidus, which should not be confused with diabetes mellitus. **4. Normally insulin binds to special receptors sites on the cell and initiates a series of reactions involved in metabolism. In type 2 diabetes, these reactions are diminished primarily as a result of obesity and aging.**

The nurse caring for a client diagnosed with cancer of the pancreas writes the collaborative problem of "altered nutrition." Which intervention should the nurse include in the plan of care? 1. Continuous feedings via PEG tube. 2. Have the family bring in foods from home. 3. Assess for food preferences. 4. Refer to the dietitian.

4 1. Tube feedings are collaborative interventions, but the stem did not say the client had a feeding tube. 2. This is an independent intervention. 3. Assessment is an independent intervention and the first step in the nursing process. No one should have to tell the nurse to assess the client. ***4. A collaborative intervention would be to refer to the nutrition expert, the dietitian.** TEST-TAKING HINT: The key word in the stem is "collaborative." which means another healthcare discipline must be involved. Only options "1" and "4" involve other members of the health-care team. The test taker could eliminate distracter "1" by rereading the stem and realizing that the stem did not say the client had a feeding tube.

The nurse identified a concept of metabolism for a client diagnosed with diabetes mellitus type 1. Which interventions should the nurse include in the plan of care? Select all that apply. 1. Teach the client to perform self glucose monitoring. 2. Instruct the client about complications of high-glucose levels. 3. Instruct the client to inspect the feet daily. 4. Explain the need to carry a source of quick-acting proteins. 5. Encourage the client to have regular eye exams.

ANS: 1, 2, 3, 5 **1. The client with diabetes should be taught to perform self glucose monitoring.** **2. In order to maintain a healthy lifestyle the client should be aware of the consequences of not controlling the blood glucose.** **3. Diabetes affects all tissues in the body. The feet are particularly at risk for the development of foot sores.** 4. The client should carry sources of quick-acting carbohydrates, not protein. **5. Diabetes can cause retinal changes and detachment.** TEST-TAKING HINT: The nurse should be able to teach common information to the client.

The client diagnosed with type 1 diabetes mellitus received regular insulin two (2) hours ago. The client is complaining of being jittery and nervous. Which interventions should the nurse implement? List in order of priority. 1. Call the laboratory to confirm blood glucose level. 2. Administer a quick-acting carbohydrate. 3. Have the client eat a bologna sandwich. 4. Check the client's blood glucose level at the bedside. 5. Determine if the client has had anything to eat.

In order of performance: 5, 2, 4, 1, 3. 5. Regular insulin peaks in two (2) to four (4) hours; therefore, the nurse should suspect a hypoglycemic reaction if the client has not eaten anything. 2. The antidote for insulin is glucose; therefore, the nurse should give the client some type of quick-acting food source. 4. The nurse should obtain the client's blood glucose level as soon as possible; this can be done with a glucometer at the bedside. 1. Most hospitals require a confirmatory serum blood glucose level. Do not wait for results to give food. 3. A source of long-acting carbohydrate and protein should be given to prevent a reoccurrence of hypoglycemia.


Related study sets

Mental health service provision, Law, Pharma, Intellectual disability, Childhood, Personality disorders, Alcohol, Alcohol and substance abuse, Eating disorders, Anxiety and somatoform disorders, Anxiety, fear and avoidance, Schizophrenia, Psychosis,...

View Set